Selected Exercises From Abstract Algebra by Dummit and Foote (3rd Edition)
Selected Exercises From Abstract Algebra by Dummit and Foote (3rd Edition)
Selected Exercises From Abstract Algebra by Dummit and Foote (3rd Edition)
Section 8.1
Exercise 2 (a). Show that 13 is relatively prime to 20 and determine the inverse of 13 mod 20.
Solution. We use the wonderful Euclidean algorithm. See that
20 = (1)13 + 7
13 = (1)7 + 6
7 = (1)6 + 1
6 = (6)1 + 0.
Since 1 is the last non zero reminder we conclude that the gcd(13, 20) = 1 as desired. To find
the inverse of 13 modulo 20 we trace back the algorithm.
1=7−6
= 7 − (13 − 7)
= 7 − (13 − (20 − 13))
= (20 − 13) − (13 − (20 − 13))
= (2)20 − (3)13.
Taking the modulo of the last expression we see that the inverse of 13 is −3 or, equivalently,
17.
Exercise 3. Let R be a Euclidean domain. Let m be the minimum integer in the set of norms
of nonzero elements of R. Prove that every nonzero element of R of norm m is a unit. Deduce
that a nonzero element of norm zero (if such an element exist) is a unit.
Proof. Let u have minimal norm among the elements in R as described in the problem. Then,
since R is an Euclidean domain, we can write 1 as qu + r. Where N (u) > N (r) ≥ 0. By
construction, N (r) = 0 as u has minimal norm. Hence 1 = qu, and u is a unit. The second
part of the problem follows from the fact that any element of norm zero has minimal norm.
Exercise 5 (a). Determine all integer solutions of 2x + 4y = 5.
Solutions. Note that gcd(2, 4) = 2 and that 2 does not divide 5. Therefore, no solutions
exist.
√
Exercise 9. Prove that the ring of integers O in the quadratic integer ring Q( 2) is a Euclidean
√
domain with respect to the norm given by the absolute value of the field norm N (a + b 2) =
a2 − 2b2 .
1
√ √
Proof. Let α = a1 + a2 2 and β = b1 + b2 2 be any two elements in O. Then, note that
√
α a1 + a2 2
= √
β b1 + b2 2
√ √
(a1 + a2 2)(b1 − b2 2)
=
b21 − 2b22
a2 b 1 − a1 b 2 √
a1 b1 − 2a2 b2
= + 2
b1 − 2b22 b21 − 2b22
√
= d1 + d2 2.
Since the above components may not be elements in Z, we use the closest integer to each (let
them be c1 and c2 so that
α √ √
= λ + = (c1 + c2 2) + (d1 − c1 ) + (d2 − c2 ) 2
β
where α, β, and λ are elements in the ring.
From the previous argument, let β = α−λβ so that β is an element of the ring and furthermore
α = λβ + (α − λβ) = λβ + β.
It is left to show that N (β) > N (β). To that end, we show that N () < 1. Note that
Section 8.2
√ √
integer ring Z[ −5]. Define the ideals I2 = (2, 1 + −5),
Exercise 5.√Let R be the quadratic √
I3 = (3, 2 + −5), and I30 = (3, 2 − −5).
(a) Prove that I2 , I3 , and I30 are non-principal ideals in R.
Then, since all the factors above are members of Z, it must be true that a2 +5b2 divides
both 4 and 6. Therefore a2 + 5b2 is equal to 1 or 2. Note that there are no integers
2
solutions to a2 + 5b2 = 2 and therefore a2 + 5b2 is forced
√ to be equal to 1. The latter
forces a to be ±1 and b to be 0. It follows that (2, 1 + −5) = (1) is the entire ring.
In particular there exist α and β such that
√
α(2) + β(1 + −5) = 1.
√
Equivalently, multiplying by (1 − −5),
√ √
α(2)(1 − −5) + β(6) = (1 − −5).
Observe that the left hand side has a factor of 2 and if we take the norm N we have,
√ √
N (2)N (α(1 − −5) + β(3)) = N (1 − −5).
√
Therefore N (2) = 4 divides N (1 − −5) = 6, a contradiction.
√
ii) The ideal I2 = (3, 2 + −5) is not principal.
Analogous to the previous case, let a, b, α, β be as in i). Then, the following must hold
N (α)(9)(9) + N (β)(9)(9) = 9.
Rearranging,
9(N (α) + N (β)) = 1,
a contradiction since the norm is an integer valued function. It must be the case that
a2 + 5b2 = 9 and either a2 = 9 and b2 = 0, or a2 = 4 and√b2 = 1. In the former √ we
have that the ideal is generated by (±3). Furthermore
√ 2 + −5 = (±3) · (z1 + c2 −5)
is a contradiction, since 3 does not divide 2 + −5. Then, it must be the case that
a2 = 4 and√b2 = 1. Let the ideal be generated by this choice of a and b. Then
3 = (±a ± −5)(z), where z is an element of the ring. By taking the norm we get
9 = 9 · N (z). Then N (z) = 1 and z = ±1, a clear contradiction. We conclude that the
ideal is not principal.
√
iii) The ideal I2 = (3, 2 + −5) is not principal.
Analogous to the previous case, let a, b, α, β be as in i). Then, the following must hold
3
(b) Prove that the product of two non-principal ideals can be principal by showing that I22 is the
principal ideal generated by 2, i.e., I22 = (2).
√ √
Proof. Observe that I22 is the ideal (4, 2 + 2 −5, −4 + 2 −5). Furthermore,
√ √
2 = (2 + 2 −5) − (−4 + 2 −5) − 4,
hence (2) ⊂ I22 . The inclusion I22 ⊂ (2) follows form factoring out 2 from each of the
generators in the ideal.
√ √
(c) Prove similarly that I2 I3 = (1 − −5) and I2 I30 = (1 + −5) are principal. Conclude that
the principal ideal (6) is the product of 4 ideals: (6) = I22 I3 I30 .
√ √
Proof.
√ Similar to part (b), observe that the ideal I2 I3 is equal to (6, 4+2 −5, 3+3 −5, −3+
3 −5). Furthermore, since
√ √ √
1 − 5 = (4 + 2 −5) − (3 + 3 −5),
√
we have
√ the inclusion (1 − −5) ⊂ I2 I3 . The reverse inclusion comes from the fact that
1 − −5 is a factor to all the generators; i.e.
√ √
6 = (1 − −5) · (1 + −5)
√ √ √
(4 + 2 −5) = (1 − −5) · (−1 + −5)
√ √ √
(3 + 3 −5) = (1 − −5) · (−2 + −5)
√ √
(−3 + 3 −5) = (1 − −5) · (−3).
√ √ √
Analogously, I2 I30 is generated by (6, 4 − 2 −5, 3 + 3 −5, 7 + −5); we have
√ √ √
1 + −5 = (4 − 2 −5) + (3 + 3 −5) − 6;
and
√ √
−5) · (1 − −5)
6 = (1 +
√ √ √
(4 − 2 −5) = (1 + −5) · (−1 − −5)
√ √
(3 + 3 −5) = (1 + −5) · (3)
√ √ √
(7 + −5) = (1 + −5) · (2 − −5).
√
Therefore I2 I30 = (1+ −5). Furthermore I2 I3 I2 I30 = (6), and since the ring is commutative
we have I22 I3 I30 = (6) as desired.
Section 8.3
√
Exercise 5. Let R = Z[ −n] where n is a squarefree integer greater than 3.
√ √
(a) Prove that 2, −n and 1 + −n are irreducibles in R.
4
(a) 2 is irreducible.
We proceed by contradiction and assume √ that 2 is reducible. √Then 2 = a · b for two
non unit elements a and b. Let a = a1 +a2 −n and b = b1 +b2 −n where a1 , a2 , b1 , b2
are integers. We compute the norm to see that
(b) Prove that R is not a U.F.D. Conclude that the quadratic integer ring O is not a U.F.D.
√ D ≡ 2, 3 √mod 4, D < −3 (so also not Euclidean and not a P.I.D.). [Show that either
for
−n or 1 + −n is not prime]
(c) Give an explicit ideal in R that
√ is not principal.
√ [Using (b) consider a maximal ideal
containing the nonprime ideal ( −n) or (1 + −n).]
√
Exercise
√ 8 a). Let
√ R be the quadratic integer
√ ring Z[ −5] and define ideals I2 = (2, 1 +
0
−5), I3 = (3, 2 + √−5), and I3 =√(3, 1 − −5).
Prove that 2, 3, 1 + −5 √ and 1 − −5 √ are irreducibles in R, no two of which are associate in
R, and that 6 = 2 · 3 = (1 −5) · (1 − −5) are two distinct factorizations of 6 into irreducibles
in R.
5
Proof. We inspect each element separately.
1. 2 is irreducible.
We proceed by contradiction and assume√ that 2 is reducible. √Then 2 = a · b for two non
unit elements a and b. Let a = a1 + a2 −5 and b = b1 + b2 −5 where a1 , a2 , b1 , b2 are
integers. We compute the norm to see that
Therefore (a21 + 5a22 ) divides 4 so it is either 1, 2, 4. If it is equal to 1 then the only solution
for a is ±1, a contradiction since a is not a unit. If a21 + 5a22 = 4, then b21 + 5b22 = 1, and
analogous to the previous statement b = ±1 is a unit (a contradiction). The last case is
a21 + 5a22 = 2, but this has no solution for a1 , a2 ∈ Z. We conclude that 2 is irreducible.
2. 3 is irreducible
We proceed by contradiction and assume√ that 3 is reducible. √Then 3 = a · b for two non
unit elements a and b. Let a = a1 + a2 −5 and b = b1 + b2 −5 where a1 , a2 , b1 , b2 are
integers. We compute the norm to see that
Therefore (a21 + 5a22 ) divides 9 so it is either 1, 3, 9. If it is equal to 1 then the only solution
for a is a = ±1, a contradiction since a is not a unit. If a21 + 5a22 = 9, then b21 + 5b22 = 1,
and analogous to the previous statement b = ±1 is a unit (a contradiction). The last case
is a21 + 5a22 = 3, but this has no solution for a1 , a2 ∈ Z. We conclude that 3 is irreducible.
√
3. 1 + −5 is irreducible √ √
We proceed by contradiction and assume that 1 + −5 √ is reducible. Then 1 +√ −5 = a · b
for two non unit elements a and b. Let a = a1 + a2 −5 and b = b1 + b2 −5 where
a1 , a2 , b1 , b2 are integers. We compute the norm to see that